How Do You Test a Hypothesis Without Sample Variance?

Click For Summary
To test the hypothesis H0: λ=1 against Ha: λ=4 with a mean of X̄100=1.5, the significance level is set at 3%. The p-value calculation requires the sample standard deviation, which is missing, raising concerns about the feasibility of the solution. If the data points X1, X2, ..., X100 are independent and identically distributed Poisson random variables, the variance can be derived from the mean, eliminating the need for the sample variance. Understanding the relationship between the mean and variance in Poisson distributions is crucial for accurate hypothesis testing.
GabrielN00

Homework Statement


Given ##X_1,\dots,X_{100}##, test ##H_0: \lambda=1## against ##H_a: \lambda=4##. The mean ##\bar{X_{100}}=1.5##
(1) Take the decision on 3% level.
(2) Find the p-value

Homework Equations



##t=\frac{\bar{x}-\mu}{s/\sqrt{n}}##

The Attempt at a Solution



The level of significance is ##0.033##. The p-value I need to evaluate ##\frac{\bar{x}-\mu}{s/\sqrt{n}}##, but I am missing ##s##. Is it still possible to have a solution?
 
Last edited by a moderator:
Physics news on Phys.org
Is that the full problem statement?

Is the Poisson distribution mentioned somewhere?
 
GabrielN00 said:

Homework Statement


Given ##X_1,\dots,X_{100}##, test ##H_0: \lambda=1## against ##H_a: \lambda=4##. The mean ##\bar{X_{100}}=1.5##
(1) Take the decision on 3% level.
(2) Find the p-value

Homework Equations



##t=\frac{\bar{x}-\mu}{s/\sqrt{n}}##

The Attempt at a Solution



The level of significance is ##0.033##. The p-value I need to evaluate ##\frac{\bar{x}-\mu}{s/\sqrt{n}}##, but I am missing ##s##. Is it still possible to have a solution?
Do you mean that the upper limit on the type-I error is 3%? How did 3% become 0.033?

Are ##X_1, X_2, \ldots, X_{100}## independent and identically distributed? Are they Poisson random variables? If they are Poisson, you can use the formula for the variance of a Poisson to get the exact standard deviation ##\sigma##, so there is no need to use the unavailable sample variance, ##s##. (However, for a Poisson, variance is a function of the mean, so be careful).
 
Question: A clock's minute hand has length 4 and its hour hand has length 3. What is the distance between the tips at the moment when it is increasing most rapidly?(Putnam Exam Question) Answer: Making assumption that both the hands moves at constant angular velocities, the answer is ## \sqrt{7} .## But don't you think this assumption is somewhat doubtful and wrong?

Similar threads

  • · Replies 7 ·
Replies
7
Views
2K
  • · Replies 10 ·
Replies
10
Views
2K
  • · Replies 1 ·
Replies
1
Views
1K
  • · Replies 8 ·
Replies
8
Views
2K
  • · Replies 7 ·
Replies
7
Views
3K
  • · Replies 20 ·
Replies
20
Views
3K
  • · Replies 5 ·
Replies
5
Views
2K
  • · Replies 1 ·
Replies
1
Views
2K
  • · Replies 2 ·
Replies
2
Views
2K
  • · Replies 8 ·
Replies
8
Views
1K